Tải bản đầy đủ (.pdf) (18 trang)

MỘT SỐ BÀI TOÁN SỐ HỌC LIÊN QUAN ĐẾN LŨY THỪA

Bạn đang xem bản rút gọn của tài liệu. Xem và tải ngay bản đầy đủ của tài liệu tại đây (406.84 KB, 18 trang )

MỘT SỐ BÀI TOÁN SỐ HỌC
LIÊN QUAN ĐẾN LŨY THỪA
Phạm Văn Quốc
(Trường THPT chuyên Khoa Học Tự Nhiên)
Trong các kỳ thi học sinh giỏi chúng ta hay gặp các bài toán số học liên quan
đến lũy thừa như chứng minh sự chia hết, chứng minh sự tồn tại hoặc tìm các
số nguyên thỏa mãn điều kiện, Trong những năm gần đây, dạng toán này cũng
xuất hiện nhiều trong các đề thi quốc gia, đề thi chọn đội tuyển thi quốc tế
(CĐT) của các nước, các đề dự tuyển và các đề thi Toán quốc tế (IMO). Đây là
những bài toán hay và tất nhiên không dễ nếu không nắm được một số kỹ thuật
cũng như nhận dạng được kiểu bài toán. Các lời giải thường sử dụng công cụ
không khó nhưng chứa đựng nhiều sự tinh tế và sự linh hoạt trong vận dụng kiến
thức. Bài viết dưới đây đề cập đến một số kiến thức cơ bản và kỹ năng liên quan
đến các bài toán dạng này.
I. Kiến thức cơ bản
Trong phần này là một số kiến thức cơ bản nhưng chúng hay được dùng trong
các dạng toán mà ta đang xét: công thức lũy thừa, số mũ "đúng", định lý Fermat,
Định lý Euler, cấp của số nguyên và một số tính chất liên quan hay dùng .
1. Một số khai triển liên quan đến lũy thừa
Định lý 1. Cho 𝑛 là số nguyên dương, khi đó với 𝑥, 𝑦 bất kỳ ta có
∘ 𝑥
𝑛
− 𝑦
𝑛
= (𝑥 − 𝑦) (𝑥
𝑛−1
+ 𝑥
𝑛−2
𝑦 + 𝑥
𝑛−3
𝑦


2
+ · · · + 𝑥𝑦
𝑛−2
+ 𝑦
𝑛−1
) ,
∘ 𝑥
𝑛
+ 𝑦
𝑛
= (𝑥 + 𝑦) (𝑥
𝑛−1
− 𝑥
𝑛−2
𝑦 + 𝑥
𝑛−3
𝑦
2
− · · · − 𝑥𝑦
𝑛−2
+ 𝑦
𝑛−1
) nếu 𝑛 lẻ,
∘ (𝑥 + 𝑦)
𝑛
= 𝑥
𝑛
+ 𝐶
1
𝑛

𝑥
𝑛−1
𝑦 + 𝐶
2
𝑛
𝑥
𝑛−2
𝑦
2
+ · · · + 𝐶
𝑛−1
𝑛
𝑥𝑦
𝑛−1
+ 𝑦
𝑛
.
Ta hãy bắt đầu bằng ví dụ sau.
Ví dụ 1. (Romania 2002) Cho 𝑘, 𝑛 là các số nguyên dương với 𝑛 > 2. Chứng
minh rằng phương trình
𝑥
𝑛
− 𝑦
𝑛
= 2
𝑘
không có nghiệm nguyên dương.
Lời giải.
Giả sử phương trình có nghiệm nguyên dương (𝑥, 𝑦). Nếu gcd (𝑥, 𝑦) = 𝑑 >
1 ⇒ 𝑑 | 2

𝑘
nên 𝑑 là lũy thừa của 2. Bằng cách chia hai vế cho 𝑑
𝑛
, ta có thể giả sử
gcd (𝑥, 𝑦) = 1 và suy ra 𝑥, 𝑦 lẻ.
1
www.VNMATH.com
Nếu 𝑛 chẵn, 𝑛 = 2𝑚 ta có 𝑥
𝑛
− 𝑦
𝑛
= (𝑥
𝑚
− 𝑦
𝑚
) (𝑥
𝑚
+ 𝑦
𝑚
) nên 𝑥
𝑚
− 𝑦
𝑚
=
2
𝑎
, 𝑥
𝑚
+ 𝑦
𝑚

= 2
𝑘−𝑎
với 𝑎 là số nguyên dương. Khi đó 𝑥
𝑚
= 2
𝑎−1

1 + 2
𝑘−2𝑎


𝑥 lẻ nên 𝑎 = 1. Hơn nữa vì 𝑚 ≥ 2 nên
𝑥
𝑚
− 𝑦
𝑚
= (𝑥 − 𝑦)

𝑥
𝑚−1
+ 𝑥
𝑚−2
𝑦 + · · · + 𝑦
𝑚−1

> 2
mâu thuẫn.
Do đó 𝑛 là số lẻ. Ta có
𝑥
𝑛

− 𝑦
𝑛
= (𝑥 − 𝑦)

𝑥
𝑛−1
+ 𝑥
𝑛−2
𝑦 + · · · + 𝑦
𝑛−1

.
Nhưng do 𝑥, 𝑦 lẻ nên 𝑥
𝑛−1
+ 𝑥
𝑛−2
𝑦 + · · · + 𝑦
𝑛−1
≡ 𝑛 ≡ 1 (mo d 2). Suy ra 𝑥
𝑛−1
+
𝑥
𝑛−2
𝑦 + · · ·+ 𝑦
𝑛−1
= 1, điều này là không thể vì 𝑥, 𝑦 nguyên dương và 𝑛 > 2. Vậy
phương trình đã cho vô nghiệm. 
Lời giải của ví dụ này chủ yếu dùng công thức của hiệu hai lũy thừa và tính
chất tích của hai số nguyên dương là lũy thừa của 2 thì mỗi số là lũy thừa của 2.
Ví dụ tiếp theo cũng có ý giải cũng gần giống nhưng cần một chút khéo léo hơn.

Ví dụ 2. (Dự tuyển IMO 2008) Cho 𝑛 là số nguyên dương và 𝑝 là số nguyên tố.
Chứng minh rằng nếu 𝑎, 𝑏, 𝑐 là các số nguyên (không nhất thiết dương) thỏa mãn
đẳng thức
𝑎
𝑛
+ 𝑝𝑏 = 𝑏
𝑛
+ 𝑝𝑐 = 𝑐
𝑛
+ 𝑝𝑎
thì 𝑎 = 𝑏 = 𝑐.
Lời giải.
Ta chứng minh bài toán bằng phản chứng. Rõ ràng nếu hai trong ba số 𝑎, 𝑏, 𝑐
bằng nhau thì tất cả chúng bằng nhau. Giả sử cả ba số phân biệt đôi một, khi
đó theo giả thiết ta có
𝑎
𝑛
− 𝑏
𝑛
𝑎 − 𝑏
.
𝑏
𝑛
− 𝑐
𝑛
𝑏 − 𝑐
.
𝑐
𝑛
− 𝑎

𝑛
𝑐 − 𝑎
= −𝑝
3
.
Vì vế phải âm nên có ít nhất một trong ba số 𝑎, 𝑏, 𝑐 là số âm. Hơn nữa 𝑛 phải là
số chẵn (nếu trái lại suy ra mỗi thừa số ở vế trái là số dương).
+, Nếu 𝑝 là số lẻ, mà
2 
𝑎
𝑛
− 𝑏
𝑛
𝑎 − 𝑏
= 𝑎
𝑛−1
+ 𝑎
𝑛−2
𝑏 + · · · + 𝑏
𝑛−1
.
Suy ra 𝑎, 𝑏 khác tính chẵn lẻ, tức là 2  𝑎 − 𝑏. Tương tự 2  𝑏 − 𝑐, 2  𝑐 − 𝑎 đây là
điều mâu thuẫn.
+, Nếu 𝑝 = 2. Rõ ràng nếu một trong ba số 𝑎, 𝑏, 𝑐 bằng 0 thì cả ba số bằng 0.
Xét |𝑎| , |𝑏| , |𝑐| ≥ 1. Đặt 𝑛 = 2𝑚 ta có
𝐴 =
𝑎
𝑛
− 𝑏
𝑛

𝑎 − 𝑏
= (𝑎 + 𝑏)
𝑎
2𝑚
− 𝑏
2𝑚
𝑎
2
− 𝑏
2
2
www.VNMATH.com
= (𝑎 + 𝑏)

𝑎
2𝑚−2
+ 𝑎
2𝑚−4
𝑏
2
+ · · · + 𝑏
2𝑚−2

.
Dễ thấy nếu 𝑚 > 1 ta có ngay |𝐴| ≥ 4 (do 𝐴 ̸= 0) và tương tự suy ra mâu thuẫn
vì tích của chúng là −8. Do đó 𝑚 = 1 ⇒ 𝑛 = 2 và ta thu được
(𝑎 + 𝑏) (𝑏 + 𝑐) (𝑐 + 𝑎) = −8.
Chú ý là do 𝑝 = 2 nên từ giả thiết ta có ngay 𝑎, 𝑏, 𝑐 cùng tính chẵn lẻ, suy ra
𝑎 + 𝑏, 𝑏 + 𝑐, 𝑐 + 𝑎 chẵn. Mà −8 = 2.2. (−2) nên dễ thấy hai trong ba số 𝑎, 𝑏, 𝑐 phải
bằng nhau, suy ra 𝑎 = 𝑏 = 𝑐 (mâu thuẫn). Từ đó ta có điều phải chứng minh. 

2. Số mũ "đúng" (exact exponent)
Định nghĩa 2. Cho 𝑝 là số nguyên tố, 𝑎 là số nguyên và 𝛼 là số tự nhiên. Ta nói
𝑝
𝛼
là ước đúng (exact power) của 𝑎, và 𝛼 được gọi là số mũ đúng của 𝑝 trong khai
triển của 𝑎, nếu 𝑝
𝛼
| 𝑎 và 𝑝
𝛼+1
 𝑎. Khi đó ta viết 𝑝
𝛼
‖ 𝑎 và ký hiệu 𝛼 = 𝑣
𝑝
(𝑎).
Tính chất 3. Cho 𝑎, 𝑏 là các số nguyên, khi đó ta có
∘ Nếu 𝑝
𝛼
‖ 𝑎 và 𝑝
𝛽
‖ 𝑏 thì 𝑝
𝛼+𝛽
‖ 𝑎𝑏,
∘ Nếu 𝑝
𝛼
‖ 𝑎 thì 𝑝
𝑘𝛼
‖ 𝑎
𝑘
,
∘ Nếu 𝑝

𝛼
‖ 𝑎 và 𝑝
𝛽
‖ 𝑏 với 𝑎 ̸= 𝑏 thì 𝑝
min(𝛼,𝛽)
‖ 𝑎 + 𝑏.
Ví dụ 3. Cho 𝑎, 𝑘 là các số nguyên dương và 𝑝 là số nguyên tố lẻ, 𝛼 ∈ N
+
sao
cho 𝑝
𝛼
‖ 𝑎 − 1, khi đó với mọi số nguyên 𝛽 ≥ 0 thì 𝑝
𝛼+𝛽
‖ 𝑎
𝑘
− 1 ⇔ 𝑝
𝛽
‖ 𝑘.
Lời giải.
Ta chứng minh bằng quy nạp theo 𝛽. Nếu 𝛽 = 0, thì
𝑎
𝑘
− 1
𝑎 − 1
= 𝑎
𝑘−1
+ · · · + 𝑎 + 1 ≡ 𝑘 (mod 𝑝) do 𝑎 ≡ 1 (mod 𝑝)
và suy ra nó không chia hết cho 𝑝. Bài toán đúng.
Giả sử bài toán đúng với 𝛽 ≥ 0 nào đó và 𝑘 = 𝑝
𝛽+1

𝑡 với 𝑝  𝑡. Theo giả thiết
quy nạp ta có 𝑎
𝑘/𝑝
= 𝑎
𝑝
𝛽
𝑡
= 𝑚𝑝
𝛼+𝛽
+ 1 với 𝑚 không chia hết cho 𝑝. Hơn nữa ta

𝑎
𝑘
− 1 =

𝑚𝑝
𝛼+𝛽
+ 1

𝑝
− 1
=

𝑚𝑝
𝛼+𝛽

𝑝
+ · · · + 𝐶
2
𝑝


𝑚𝑝
𝛼+𝛽

2
+ 𝑚𝑝
𝛼+𝛽+1
.
Vì 𝑝 | 𝐶
2
𝑝
=
𝑝 (𝑝 − 1)
2
, nên tất cả các số hạng trong khai triển trên, ngoại trừ số
hạng cuối cùng, đều chia hết cho 𝑝
𝛼+𝛽+2
. Từ đó ta có ngay 𝑝
𝛼+𝛽+1
‖ 𝑎
𝑘
− 1. Theo
nguyên lý quy nạp ta có điều phải chứng minh. 
Tương tự như ví dụ trên, ta có bài toán với 𝑝 = 2. Trong trường hợp này do
𝐶
2
𝑝
= 1 nên bài toán có sự thay đổi một chút.
Ví dụ 4. Cho 𝑎, 𝑘 là các số nguyên dương, 𝛼 ∈ N
+

sao cho 2
𝛼
‖ 𝑎
2
− 1, khi đó
với mọi số nguyên 𝛽 ≥ 0 thì 2
𝛼+𝛽
‖ 𝑎
𝑘
− 1 ⇔ 2
𝛽+1
‖ 𝑘.
3
www.VNMATH.com
Lời giải.
Rõ ràng ta chỉ cần xét với 𝑘 là số nguyên dương chẵn. Ta chứng minh bài
toán bằng quy nạp theo 𝛽 ≥ 0. Với 𝛽 = 0 ⇔ 𝑘 = 2𝑛 (𝑛 là số lẻ). Khi đó
𝑎
𝑘
− 1
𝑎
2
− 1
=
𝑛−1

𝑖=0
𝑎
2(𝑛−1−𝑖)
≡ 𝑛 ̸≡ 0 (mod 2

𝛼
) .
Bài toán đúng với 𝛽 = 0. Giả sử bài toán đúng với 𝛽 ≥ 0, đặt 𝑘 = 2𝑛 với 2
𝛽+1
‖ 𝑛,
khi đó
𝑎
𝑘
− 1
𝑎
𝑛
− 1
= 𝑎
𝑛
+ 1 ≡ 2 (mod 4)
(do 𝑎 lẻ và 𝑛 chẵn). Mà 2
𝛼+𝛽
‖ 𝑎
𝑛
− 1. Suy ra bài toán đúng đến 𝛽 + 1. Ta có
điều phải chứng minh. 
Chú ý: Hai ví dụ trên có thể tổng quát hơn, đó là định lý về số mũ đúng như
dưới đây, cách chứng minh hoàn toàn tương tự bằng quy nạp.
Định lý 4. (Lifting the Exponent Lemma) i. Với 𝑥, 𝑦 là các số nguyên (không
nhất thiết dương), 𝑛 là số nguyên dương và 𝑝 là số nguyên tố lẻ sao cho 𝑝 | 𝑥 − 𝑦
và 𝑥, 𝑦 không chia hết cho 𝑝. Khi đó
𝑣
𝑝
(𝑥
𝑛

− 𝑦
𝑛
) = 𝑣
𝑝
(𝑥 − 𝑦) + 𝑣
𝑝
(𝑛) .
ii. Với 𝑥, 𝑦 là hai số nguyên lẻ và 𝑛 là số nguyên dương chẵn. Khi đó
𝑣
2
(𝑥
𝑛
− 𝑦
𝑛
) = 𝑣
2
(𝑥 − 𝑦) + 𝑣
2
(𝑥 + 𝑦) + 𝑣
2
(𝑛) − 1.
Nhận xét: +, Nếu 𝑛 lẻ bằng cách thay 𝑦 bởi −𝑦 thì ta cũng có đẳng thức tương
tự trong phần i, 𝑣
𝑝
(𝑥
𝑛
+ 𝑦
𝑛
) = 𝑣
𝑝

(𝑥 + 𝑦) + 𝑣
𝑝
(𝑛) ;
+, Trong phần ii, do 𝑥, 𝑦 lẻ nên một trong hai số 𝑣
2
(𝑥 − 𝑦) , 𝑣
2
(𝑥 + 𝑦) bằng
1.
Ví dụ 5. (Nga 1996) Các số nguyên dương 𝑎, 𝑏, 𝑝, 𝑛, 𝑘 thỏa mãn 𝑎
𝑛
+ 𝑏
𝑛
= 𝑝
𝑘
.
Chứng minh rằng nếu 𝑛 > 1 là số lẻ và 𝑝 là số nguyên tố lẻ thì 𝑛 là lũy thừa của
𝑝.
Lời giải.
Ta có 𝑝
𝑘
= (𝑎 + 𝑏) (𝑎
𝑛−1
− 𝑎
𝑛−2
𝑏 + · · · + 𝑏
𝑛−1
) suy ra 𝑎 + 𝑏 = 𝑝
𝑗
, 𝑗 ≥ 1. Giả sử

𝑝
𝑙
‖ 𝑛 mà 𝑎 ≡ −𝑏 (mod 𝑝). Nên theo định lý 4 ta có
𝑣
𝑝
(𝑎
𝑛
+ 𝑏
𝑛
) = 𝑣
𝑝
(𝑎 + 𝑏) + 𝑣
𝑝
(𝑛) = 𝑗 + 𝑙,
tức là 𝑝
𝑙+𝑗
‖ (𝑎
𝑛
+ 𝑏
𝑛
) = 𝑝
𝑘
⇒ 𝑙 = 𝑘 − 𝑗. Hơn nữa theo định lý 4 ta lại có
𝑣
𝑝

𝑎
𝑝
𝑘−𝑗
+ 𝑏

𝑝
𝑘−𝑗

= 𝑣
𝑝
(𝑎 + 𝑏) + 𝑣
𝑝

𝑝
𝑘−𝑗

= 𝑗 + 𝑘 − 𝑗 = 𝑘,
4
www.VNMATH.com
suy ra 𝑝
𝑘
‖ 𝑎
𝑝
𝑘−𝑗
+ 𝑏
𝑝
𝑘−𝑗
và 𝑎
𝑝
𝑘−𝑗
+ 𝑏
𝑝
𝑘−𝑗
‖ 𝑎
𝑛

+ 𝑏
𝑛
(vì 𝑛 lẻ và 𝑝
𝑘−𝑗
‖ 𝑛). Mà
𝑎
𝑛
+ 𝑏
𝑛
= 𝑝
𝑘
nên ta có 𝑝
𝑘
= 𝑎
𝑝
𝑘−𝑗
+ 𝑏
𝑝
𝑘−𝑗
= 𝑎
𝑛
+ 𝑏
𝑛
và từ đó 𝑛 = 𝑝
𝑘−𝑗
. 
3. Định lý Fermat (nhỏ)
Định lý 5. (Định lý Fermat) Cho 𝑝 là một số nguyên tố thỏa mãn 𝑝  𝑎. Khi đó
𝑎
𝑝−1

≡ 1 (mod 𝑝) .
Đối với số nguyên 𝑎 bất kỳ, ta có 𝑎
𝑝
≡ 𝑎 (mod 𝑝) .
Ví dụ 6. Giả sử số nguyên tố 𝑝 có dạng 4𝑘 + 3 và là ước của (𝑥
2
+ 𝑦
2
) thì
𝑝 | 𝑥, 𝑝 | 𝑦.
Lời giải.
Giả sử trái lại 𝑝  𝑥, 𝑝  𝑦 ⇒ gcd (𝑥, 𝑝) = gcd (𝑦, 𝑝) = 1. Do đó theo định
lý Fermat ta có 𝑥
𝑝−1
≡ 𝑦
𝑝−1
≡ 1 (mod 𝑝). Trong khi đó từ giả thiết ta có
𝑥
2
≡ −𝑦
2
(mod 𝑝) ⇒ (𝑥
2
)
𝑝−1
2
≡ (−𝑦
2
)
𝑝−1

2
(mod 𝑝). Hay 𝑥
𝑝−1
≡ −𝑦
𝑝−1
(mod 𝑝) ⇔
1 ≡ −1 (mod 𝑝). Ta có mâu thuẫn vì 𝑝 là số nguyên tố lẻ. 
Nhận xét: Từ ví dụ này ta suy ra ngay với 𝑛 là số nguyên dương mọi ước nguyên
tố lẻ của 𝑛
2
+ 1 đều có dạng 4𝑘 + 1. Và từ đó mọi ước dương của 𝑛
2
+ 1 đều có
dạng 2
𝑖
(4𝑘 + 1) , 𝑖 = 0, 1.
Ví dụ 7. (CĐT Mỹ 2008) Tìm tất cả các số nguyên dương 𝑛 sao cho 𝑛
7
+ 7 là
số chính phương.
Lời giải.
Giả sử tồn tại 𝑚 ∈ N sao cho 𝑛
7
+ 7 = 𝑚
2
. Khi đó ta có
𝑚
2
+ 11
2

= 𝑛
7
+ 2
7
= (𝑛 + 2)

𝑛
6
− 2𝑛
5
+ · · · − 2
5
𝑛 + 2
6

.
Rõ ràng 𝑚
2
+ 11
2
≡ 1, 2 (mod 4) ⇒ 𝑛
7
+ 2
7
≡ 1, 2 (mod 4). Từ đó dễ dàng thấy
𝑛 ≡ 1 (mod 4). Mà theo đẳng thức trên ta có (𝑛 + 2) | 𝑚
2
+ 11
2
suy ra 𝑚

2
+ 11
2
phải có một ước nguyên tố 𝑝 có dạng 4𝑖 + 3. Từ đó 11
2
≡ −𝑚
2
(mod 𝑝). Nếu
𝑝 ̸= 11 thì theo định lý Fermat ta có 11
𝑝−1
≡ 𝑚
𝑝−1
≡ 1 (mod 𝑝) nhưng trong khi
đó
11
𝑝−1


−𝑚
2

𝑝−1
2
= −𝑚
𝑝−1
(mod 𝑝)
mâu thuẫn. Suy ra 𝑝 = 11. Do đó 𝑚 = 11𝑘, 𝑘 ∈ N và 11 | (𝑛 + 2) .
Hơn nữa từ đó ta lại có
𝑛
6

− 2𝑛
5
+ · · · − 2
5
𝑛 + 2
6
≡ 7.2
6
≡ 8 (mod 11) .
Suy ra 11
2
| (𝑛 + 2) ⇒ 𝑛 = 11
2
ℎ − 2 và ℎ là ước dương của 𝑘
2
+ 1. Theo ví
dụ 6 các ước nguyên tố lẻ của 𝑘
2
+ 1 chỉ có dạng 4𝑖 + 1, tức là ℎ chỉ có dạng
2
𝑗
(4𝑖 + 1) , 𝑗 = 0, 1. Khi đó 𝑛 = 11
2
ℎ − 2 ≡ 0; 3 (mod 4). Ta có mâu thuẫn vì
𝑛 ≡ 1 (mod 4). Do đó không tồn tại 𝑛 thỏa mãn đầu bài. 
5
www.VNMATH.com
4. Hàm Euler
Định nghĩa 6. Cho 𝑛 là số nguyên dương. Hàm Euler 𝜙 xác định trên tập các
số nguyên dương như sau: 𝜙 (𝑛) là số các số nguyên dương nhỏ hơn 𝑛 và nguyên

tố cùng nhau với 𝑛. Quy ước 𝜙 (1) = 1.
Tính chất 7. Hàm 𝜙 (𝑛) có tính chất nhân theo nghĩa: Nếu 𝑎, 𝑏 là hai số nguyên
tố cùng nhau thì 𝜙 (𝑎𝑏) = 𝜙 (𝑎) 𝜙 (𝑏) .
Tính chất 8. Giả sử 𝑛 = 𝑝
𝛼
1
1
𝑝
𝛼
2
2
. . . 𝑝
𝛼
𝑘
𝑘
là phân tích tiêu chuẩn của 𝑛 > 1. Khi
đó
𝜙 (𝑛) = 𝑛

1 −
1
𝑝
1

1 −
1
𝑝
2

· · ·


1 −
1
𝑝
𝑘

.
Định lý 9. (Định lý Euler) Cho 𝑎, 𝑛 là các số nguyên, 𝑛 > 1, (𝑎, 𝑛) = 1. Khi đó
𝑎
𝜙(𝑛)
≡ 1 (mod 𝑛) .
Với 𝑎, 𝑛 là hai số nguyên dương bất kỳ ta có
𝑎
𝑛
≡ 𝑎
𝑛−𝜙(𝑛)
(mod 𝑛) .
Nhận xét: Định lý Fermat là trường hợp riêng của định lý Euler trong trường
hợp 𝑛 là số nguyên tố.
Bổ đề 10. Với 𝑎, 𝑏 là các số nguyên dương 𝑎, 𝑏 thì với 𝑛 đủ lớn ta có
𝑏
𝑛+𝜙(𝑎)
≡ 𝑏
𝑛
(mod 𝑎)
(cụ thể hơn 𝑛 ≥ max

𝑣
𝑝
𝑖

(𝑎) + 1, 𝑝
𝑖
là ước nguyên tố của 𝑎

).
Chứng minh của bổ đề này có thể xem trong bài 7 dưới đây.
Ví dụ 8. (IMO 1971) Chứng minh rằng dãy số 2
𝑛
− 3 (𝑛 > 1) chứa một dãy con
vô hạn gồm các số đôi một nguyên tố cùng nhau.
Lời giải.
Ta xây dựng dãy con bằng quy nạp. Giả sử ta đã xây dựng được dãy các số
𝑎
1
= 2
𝑛
1
− 1, 𝑎
2
= 2
𝑛
2
− 1, , 𝑎
𝑘
= 2
𝑛
𝑘
− 1, ở đó 2 = 𝑛
1
< 𝑛

2
< · · · < 𝑛
𝑘
, mà các
phần tử đôi một nguyên tố cùng nhau. Ta sẽ xây dựng 𝑎
𝑘+1
= 2
𝑛
𝑘+1
− 3 như sau:
Đặt 𝑠 = 𝑎
1
𝑎
2
𝑎
𝑘
ta có 𝑠 là số lẻ nên theo định lý Euler ta có 𝑠 | 2
𝜙(𝑠)
− 1 ⇒
2
𝜙(𝑠)
− 1 = 𝑞𝑠, 𝑞 ∈ N. Khi đó ta có 2
𝜙(𝑠)+2
− 3 = 4𝑞𝑠 − 1 là nguyên tố cùng
nhau với 𝑠, nên ta có thể chọn 𝑛
𝑘+1
= 𝜙 (𝑠) + 2. Rõ ràng 𝑛
𝑘+1
> 𝑛
𝑘

và dãy
số 𝑎
1
, 𝑎
2
, , 𝑎
𝑘
, 𝑎
𝑘+1
đôi một nguyên tố cùng nhau, từ đó ta có điều phải chứng
minh. 
Ví dụ 9. (Mỹ 1991) Chứng minh rằng với mọi số nguyên 𝑛 ≥ 1, dãy số sau
2, 2
2
, 2
2
2
, 2
2
2
2
, . . . (mod 𝑛)
là dãy hằng số kể từ một lúc nào đó.
6
www.VNMATH.com
Lời giải.
Ta chứng minh bằng quy nạp theo 𝑛. Với 𝑛 = 1 bài toán hiển nhiên đúng.
Giả sử bài toán đúng đến 𝑘 − 1, (𝑘 ≥ 2).
+, Nếu 𝑘 chẵn, 𝑘 = 2
𝑎

𝑏 với 𝑏 lẻ. Khi đó theo giả thiết quy nạp thì dãy đã cho
là hằng số từ lúc nào đó mô-đun 𝑏. Mà rõ ràng dãy này là dãy 0 mô-đun 2
𝑎
kể từ
lúc nào đó. Vì thế theo mô-đun 𝑘 nó cũng là dãy hằng số kể từ một lúc nào đó.
+, Nếu 𝑘 lẻ, theo định lý Euler ta có 2
𝜙(𝑘)
≡ 1 (mod 𝑘). Mà theo giả thiết quy
nạp dãy 1, 2, 2
2
, 2
2
2
, . . . (dãy các số mũ) là hằng số kể từ một lúc nào đó mô-đun
𝜙 (𝑘). Vì thế 2, 2
2
, 2
2
2
, 2
2
2
2
, . . . (mod 𝑘) là dãy hằng số kể từ một lúc nào đó.
Theo nguyên lý quy nạp ta có điều phải chứng minh. 
5. Cấp (order) của một số nguyên
Định nghĩa 11. Cho 𝑎, 𝑛 là hai số nguyên dương và 𝑎 là số nguyên bất kỳ thỏa
mãn (𝑎, 𝑛) = 1. Số nguyên dương ℎ nhỏ nhất sao cho 𝑎

≡ 1 (mo d 𝑛) được gọi

là cấp của 𝑎 mô-đun 𝑛, ký hiệu ord
𝑛
(𝑎) .
Bổ đề 12. Cho 𝑚, 𝑛 là các số nguyên dương và 𝑎 là số nguyên khác 1. Khi đó
gcd (𝑎
𝑚
− 1, 𝑎
𝑛
− 1) =


𝑎
gcd(𝑚,𝑛)
− 1


.
Định lý 13. Cho 𝑎, 𝑛 là các số nguyên nguyên tố cùng nhau với 𝑛 > 0 khi đó
ord
𝑛
(𝑎) | 𝜙 (𝑛).
Định nghĩa 14. Nếu 𝑟, 𝑛 là các số nguyên tố cùng nhau với 𝑛 > 0 và nếu
ord
𝑛
𝑟 = 𝜙 (𝑛) thì 𝑟 được gọi là căn nguyên thủy mô-đun 𝑛.
Định lý 15. Nếu 𝑟, 𝑛 là các số nguyên dương nguyên tố cùng nhau và nếu 𝑟 là
căn nguyên thủy mô-đun 𝑛. Khi đó các số nguyên

𝑟, 𝑟
2

, , 𝑟
𝜙(𝑛)

tạo thành một
hệ thặng dư thu gọn mô-đun 𝑛.
Định lý 16. (Sự tồn tại của căn nguyên thủy) Mọi số nguyên tố đều có căn
nguyên thủy. Tổng quát hơn: số nguyên dương 𝑛 > 1 có căn nguyên thủy khi
và chỉ khi 𝑛 = 2, 4, 𝑝
𝑘
hoặc 2𝑝
𝑘
trong đó 𝑝 là số nguyên tố lẻ và 𝑘 là số nguyên
dương.
Ví dụ 10. Cho 𝑛 là hai số nguyên dương và 𝑝 là số nguyên tố thỏa. Chứng minh
rằng nếu 𝑚 là ước nguyên tố lẻ của 𝑛
𝑝
+ 1 thì 2𝑝 | 𝑚 − 1 hoặc 𝑚 | 𝑛
2
− 1.
Lời giải.
Từ giả thiết 𝑚 | 𝑛
𝑝
+ 1 ⇒ 𝑛
𝑝
≡ −1 ̸≡ 1 (mod 𝑚) vì 𝑚 > 2. Suy ra 𝑛
2𝑝

1 (mod 𝑚).
Đặt 𝑑 = ord
𝑚

𝑛 khi đó 𝑑 là ước của 2𝑝 nhưng không là ước của 𝑝. Mà 𝑝 là số
nguyên tố suy ra 𝑑 = 2 hoặc 𝑑 = 2𝑝.
+, Nếu 𝑑 = 2 suy ra 𝑛
2
≡ 1 (mod 𝑚) hay 𝑚 | 𝑛
2
− 1.
+, Nếu 𝑑 = 2𝑝 ⇒ 2𝑝 | 𝑚 − 1 vì theo định lý Fermat ta có 𝑑 | 𝑚 −1 (đpcm). 
Ví dụ 11. (Dự tuyển IMO 2006) Tìm tất cả các nghiệm nguyên của phương
trình
𝑥
7
− 1
𝑥 − 1
= 𝑦
5
− 1.
7
www.VNMATH.com
Lời giải.
Gọi 𝑝 là một ước nguyên tố bất kỳ của
𝑥
7
− 1
𝑥 − 1
= 𝑥
6
+ 𝑥
5
+ · · · + 𝑥 + 1. Ta có

hai trường hợp:
+, Nếu 𝑝 | (𝑥 − 1), suy ra
𝑥
7
− 1
𝑥 − 1
≡ 1 + 1 + · · · + 1 + 1 ≡ 7 (mod 𝑝) ⇒ 𝑝 = 7;
+, Nếu 𝑝  (𝑥 − 1), ta có ngay ord
𝑝
𝑥 = 7. Từ đó suy ra 𝑝 ≡ 1 (mo d 7) .
Vậy ta có mọi ước nguyên dương 𝑑 của
𝑥
7
− 1
𝑥 − 1
thỏa mãn 𝑑 ≡ 0; 1 (mod 7).
Bây giờ giả sử (𝑥, 𝑦) là một nghiệm của phương trình đã cho. Mà 𝑦
5
− 1 =
(𝑦 − 1) (𝑦
4
+ 𝑦
3
+ 𝑦
2
+ 𝑦 + 1) suy ra
𝑦 − 1 ≡ 0; 1 (mo d 7)
𝑦
4
+ 𝑦

3
+ 𝑦
2
+ 𝑦 + 1 ≡ 0; 1 (mod 7) .
Ta có mâu thuẫn vì khi đó 𝑦 ≡ 1; 2 (mod 7) ⇒ 𝑦
4
+𝑦
3
+𝑦
2
+𝑦 +1 ≡ 5; 3 (mod 7).
Vậy bài toán vô nghiệm. 
Ví dụ 12. (CĐT Trung Quốc 2006) Tìm tất cả các cặp số nguyên (𝑎, 𝑛) sao cho
(𝑎 + 1)
𝑛
− 𝑎
𝑛
𝑛
là số nguyên.
Lời giải.
Xét 𝑛 ≥ 2, giả sử 𝑝 là ước nguyên tố nhỏ nhất của 𝑛. Khi đó 𝑝 | (𝑎 + 1)
𝑛
−𝑎
𝑛

(𝑝, 𝑛) = 1 nên tồn tại 𝑏 sao cho 𝑎𝑏 ≡ 1 (mod 𝑝). Ta có (𝑎 + 1)
𝑛
≡ 𝑎
𝑛
(mod 𝑝) ⇒

((𝑎 + 1) 𝑏)
𝑛
≡ 1 (mod 𝑝). Do đó 𝑑 | 𝑛 với 𝑑 = ord
𝑝
(𝑎𝑏 + 𝑏). Hơn nữa theo định lý
Fermat ta có ((𝑎 + 1) 𝑏)
𝑝−1
≡ 1 (mod 𝑝) ⇒ 𝑑 | 𝑝 − 1 ⇒ gcd (𝑑, 𝑛) = 1 (theo định
nghĩa của 𝑝). Tức là 𝑑 = 1, suy ra 𝑎 + 1 ≡ 𝑎 (mod 𝑝). Mâu thuẫn, do đó 𝑛 = 1. 
Chú ý : Ta có thể trình bày lời giải bằng căn nguyên thủy như sau: Gọi 𝑔 là
căn nguyên thủy mô-đun 𝑝. Theo giả thiết ta dễ thấy rằng 𝑎, 𝑎 + 1 không chia
hết cho 𝑝, suy ra (𝑎 + 1, 𝑝) = (𝑎, 𝑝) = 1 ⇒ 𝑎 + 1 ≡ 𝑔
𝑘
, 𝑎 ≡ 𝑔

(mod 𝑝) với 𝑘 ̸= ℎ.
Thay vào phương trình đã cho suy ra 𝑔
𝑘𝑛
≡ 𝑔
ℎ𝑛
(mod 𝑝) ⇒ 𝑝 − 1 | 𝑛 (𝑘 − ℎ). Mà
𝑝 là ước nguyên tố nhỏ nhất của 𝑛 nên (𝑝 − 1, 𝑛) = 1 ⇒ 𝑝 − 1 | 𝑘 − ℎ. Theo định
lý Fermat suy ra 𝑝 | 𝑔
𝑝−1
− 1 | 𝑔
𝑘
− 𝑔

⇒ 𝑝 = 1 mâu thuẫn. 
6. Một số hệ quả hay dùng khác

i. Nếu 𝑝 | 𝑥 thì (𝑥 + 𝑦)
𝑛
≡ 𝑛𝑥𝑦
𝑛−1
+ 𝑦
𝑛
(mod 𝑝
2
) và nếu 𝑎 ≡ 𝑏 (mod 𝑝) thì
𝑎 = 𝑘𝑝 + 𝑏 ta có 𝑎
𝑛
≡ 𝑛𝑘𝑝𝑏
𝑛−1
+ 𝑏
𝑛
(mod 𝑝
2
) .
ii . Cho 𝑚 là số nguyên dương và 𝑎, 𝑏 là hai số nguyên tố cùng nhau với 𝑚. Nếu
𝑥, 𝑦 là hai số nguyên thỏa mãn 𝑎
𝑥
≡ 𝑏
𝑥
(mod 𝑚) và 𝑎
𝑦
≡ 𝑏
𝑦
(mod 𝑚). Khi đó
𝑎
gcd(𝑥,𝑦)

≡ 𝑏
gcd(𝑥,𝑦)
(mod 𝑚).
iii . Cho 𝑝 là một số nguyên tố lẻ. Khi đó
a) nếu 𝑎 ≥ 2 thì 𝑎
𝑝
− 1 có một ước nguyên tố mà không là ước của 𝑎 − 1;
b) nếu 𝑎 ≥ 2, 𝑝 ̸= 3 hoặc 𝑎 > 2 thì 𝑎
𝑝
+ 1 có một ước nguyên tố mà không là
ước của 𝑎 + 1.
8
www.VNMATH.com
Hướng dẫn. Giả sử trái lại suy ra mọi ước của 𝐴 = 𝑎
𝑝−1
+𝑎
𝑝−2
+· · ·+𝑎+1 đều
là ước của 𝑎 − 1 mà 𝐴 = (𝑎 − 1) 𝐵 + 𝑝 ⇒ gcd (𝑎 − 1, 𝑎
𝑝−1
+ 𝑎
𝑝−2
+ · · · + 𝑎 + 1) |
𝑝 ⇒ 𝐴 là lũy thừa của 𝑝. Do đó 𝑝
2
‖ 𝑎
𝑝
− 1 ⇒ 𝐴 = 𝑝 − 1 mâu thuẫn.
iv. Nếu 𝑎 là số nguyên không chia hết cho số nguyên tố 𝑝 và có một số nguyên
dương 𝑘 thỏa mãn 𝑎

𝑘
≡ −1 (mod 𝑝) khi đó nếu 𝑑 = ord
𝑝
𝑎 thì ℎ = 𝑑/2 là số
nguyên dương nhỏ nhất thỏa mãn 𝑎

≡ −1 (mod 𝑝) .
Hướng dẫn. 𝑎
2𝑘
≡ 1 (mod 𝑝) ⇒ 𝑑 | 2𝑘 mà 𝑎
𝑘
≡ −1 (mod 𝑝) nên 𝑑 = 2ℎ ⇒

𝑎

− 1

𝑎

+ 1

≡ 0 (mod 𝑝) .
v. Nếu 𝑥 là số nguyên dương và 𝑝, 𝑞 là hai số nguyên tố sao cho 𝑞 |
𝑥
𝑝
− 1
𝑥 − 1
thì
𝑞 = 𝑝 hoặc 𝑞 ≡ 1 (mod 𝑝) .
vi. Số nguyên dương 𝑑 nhỏ nhất thỏa mãn 2

𝑑
≡ 1

mod 3
𝑘

là 𝑑 = 𝜙

3
𝑘

=
2.3
𝑘−1
. Nói cách khác 2 là căn nguyên thủy mô-đun 3
𝑛
.
Hướng dẫn. Nếu 𝑝 là số nguyên tố lẻ và 𝑟 là căn nguyên thủy mô-đun 𝑝
2
thì
𝑟 là căn nguyên thủy mô-đun 𝑝
𝑘
với mọi số nguyên dương 𝑘.
II. Bài tập áp dụng
Bài 1. (IMO 2005) Xét dãy số 𝑎
1
, 𝑎
2
, xác định bởi công thức
𝑎

𝑛
= 2
𝑛
+ 3
𝑛
+ 6
𝑛
− 1 (𝑛 = 1, 2, ) .
Xác định tất cả các số nguyên dương mà chúng nguyên tố cùng nhau với mọi số
hạng của dãy trên.
Lời giải.
Ta sẽ chứng minh với mọi số nguyên tố 𝑝 luôn tồn tại một số hạng 𝑎
𝑚
của
dãy sao cho 𝑝 | 𝑎
𝑚
, từ đó suy ra tất cả các số cần tìm là 1.
+, Với 𝑝 = 2, 3 rõ ràng 𝑎
2
= 48 từ đó 𝑝 | 𝑎
2
.
+, Xét 𝑝 > 3. Áp dụng định lý Fermat ta có
6𝑎
𝑝−2
= 3.2
𝑝−1
+ 2.3
𝑝−1
+ 6

𝑝−1
− 6 ≡ 3 + 2 + 1 − 6 ≡ 0 (mod 𝑝) .
Từ đó 𝑝 | 𝑎
𝑝−2
⇒ gcd (𝑝, 𝑎
𝑝−2
) = 𝑝 > 1. Vậy chỉ có số 1 thỏa mãn bài toán. 
Bài 2. (Dự tuyển IMO 2005) Giả sử 𝑎, 𝑏 là hai số nguyên dương sao cho 𝑎
𝑛
+ 𝑛
là ước của 𝑏
𝑛
+ 𝑛 với mọi số nguyên dương 𝑛. Chứng minh rằng 𝑎 = 𝑏.
Lời giải.
Giả sử 𝑎 ̸= 𝑏, khi đó từ giả thiết dễ thấy 𝑏 > 𝑎. Chọn 𝑝 là số nguyên tố lớn
hơn 𝑏 và lấy 𝑛 = (𝑎 + 1) (𝑝 − 1) + 1. Theo cách chọn này ta có 𝑛 ≡ 1 (mod 𝑝 − 1)
và 𝑛 ≡ −𝑎 (mod 𝑝). Khi đó theo định lý Fermat ta có
𝑟
𝑛
≡ 𝑟

𝑟
𝑝−1

𝑎+1
≡ 𝑟 (mod 𝑝) ∀𝑟 ∈ Z.
Ta lại có 𝑎
𝑛
+ 𝑛 ≡ 𝑎 − 𝑎 ≡ 0 (mod 𝑝) ⇒ 𝑝 | 𝑎
𝑛

+ 𝑛. Hơn nữa 𝑏
𝑛
+ 𝑛 ≡ 𝑏 − 𝑎 ≡
0 (mod 𝑝) ⇒ 𝑝 | 𝑏 − 𝑎. Điều này mâu thuẫn vì 𝑝 > 𝑏. Do đó 𝑎 = 𝑏 thỏa mãn. 
Bài 3. (Bulgaria 1995) Tìm tất cả các số nguyên tố 𝑝, 𝑞 sao cho 𝑝𝑞 là ước của
(5
𝑝
− 2
𝑝
) (5
𝑞
− 2
𝑞
).
9
www.VNMATH.com
Lời giải.
Do tính đối xứng nên ta có thể giả sử 𝑝 ≤ 𝑞 mà (5
𝑝
− 2
𝑝
) (5
𝑞
− 2
𝑞
) là số lẻ nên
ta có 5 ≤ 𝑝 ≤ 𝑞. Để ý rằng nếu số nguyên tố 𝑘 là ước của 5
𝑘
− 2
𝑘

thì theo định
lý Fermat ta có ngay 3 ≡ 5 − 2 ≡ 5
𝑘
− 2
𝑘
(mod 𝑘) ⇒ 𝑘 = 3.
Giả sử 𝑝 > 3, theo nhận xét trên ta có 𝑝 là ước của 5
𝑞
−2
𝑞
hay 5
𝑞
≡ 2
𝑞
(mod 𝑝).
Lại theo định lý Fermat thì 5
𝑝−1
≡ 2
𝑝−1
(mod 𝑝). Do đó
5
gcd(𝑝−1,𝑞)
≡ 2
gcd(𝑝−1,𝑞)
(mod 𝑝) .
Mà 𝑞 ≥ 𝑝 ⇒ gcd (𝑝 − 1, 𝑞) = 1 do đó ta có 5 ≡ 2 (mod 𝑝) ⇒ 𝑝 = 3 mâu thuẫn.
Suy ra 𝑝 = 3. Nếu 𝑞 > 3 suy ra 𝑞 là ước của 5
𝑝
− 2
𝑝

= 5
3
− 2
3
= 9.13 ⇒ 𝑞 = 13
thỏa mãn.
Vậy tất cả các cặp (𝑝, 𝑞) cần tìm là: (3, 3) ; (3, 13) ; (13, 3). 
Bài 4. (Dự tuyển IMO 2003) Cho 𝑝 là một số nguyên tố. Chứng minh rằng tồn
tại một số nguyên tố 𝑞 sao cho với mọi số nguyên 𝑛, số 𝑛
𝑝
− 𝑝 không chia hết cho
𝑞.
Lời giải.
Ta có
𝑝
𝑝
− 1
𝑝 − 1
= 1 + 𝑝 + 𝑝
2
+ · · · 𝑝
𝑝−1
≡ 𝑝 + 1

mod 𝑝
2

,
suy ra có ít nhất một ước nguyên tố của
𝑝

𝑝
− 1
𝑝 − 1
không đồng dư 1 mô-đun 𝑝
2
. Gọi
số nguyên tố này là 𝑞 và ta sẽ chỉ ra đây là số 𝑞 cần tìm.
Thật vậy, giả sử tồn tại số nguyên 𝑛 sao cho 𝑛
𝑝
≡ 𝑝 (mod 𝑞). Khi đó, theo
cách chọn số 𝑞 ta có 𝑛
𝑝
2
≡ 𝑝
𝑝
≡ 1 (mod 𝑞). Mặt khác, theo định lý Fermat,
𝑛
𝑞−1
≡ 1 (mod 𝑞), vì 𝑞 là số nguyên tố. Hơn nữa ta có 𝑝
2
 𝑞 − 1 nên (𝑝
2
, 𝑞 − 1) |
𝑝 ⇒ 𝑛
𝑝
≡ 1 (mod 𝑞). Suy ra 𝑝 ≡ 1 (mod 𝑞). Khi đó ta thu được
1 + 𝑝 + · · · + 𝑝
𝑝−1
≡ 𝑝 (mod 𝑞) .
Cùng với định nghĩa của 𝑞 ta có ngay 𝑝 ≡ 0 (mod 𝑞), đây là điều mâu thuẫn. Ta

có điều phải chứng minh. 
Nhận xét: Để dùng định lý Fermat, ta sẽ tìm cách chọn số 𝑞 có dạng 𝑝𝑘 + 1. Khi
đó
∃𝑛 𝑛
𝑝
≡ 𝑝 (mod 𝑞) ⇔ 𝑝
𝑘
≡ 1 (mod 𝑞) ,
tức là ∀𝑛 𝑛
𝑝
̸≡ 𝑝 (mod 𝑞) ⇔ 𝑝
𝑘
̸≡ 1 (mod 𝑞) .
Do đó để tìm 𝑞 ta sẽ chọn 𝑞 là một ước nguyên tố của 𝑝
𝑝
− 1 và ta có cách chọn
như trên.
Bài 5. (Dự tuyển IMO 2005) Tìm tất cả các số nguyên dương 𝑛 > 1 sao cho tồn
tại duy nhất số nguyên 𝑎 với 0 < 𝑎 ≤ 𝑛! thỏa mãn 𝑎
𝑛
+ 1 chia hết cho 𝑛!.
10
www.VNMATH.com
Lời giải.
Ta sẽ chỉ ra rằng tất cả các số 𝑛 cần tìm là các số nguyên tố. Thật vậy:
+, Với 𝑛 = 2 khi đó rõ ràng chỉ có duy nhất 𝑎 = 1 thỏa mãn yêu cầu.
+, Với 𝑛 > 2 và 𝑛 chẵn ta có 4 | 𝑛! nhưng 𝑎
𝑛
+1 ≡ 1, 2 (mod 4) , khi đó không
tồn tại 𝑎 thỏa mãn.

+, Xét 𝑛 lẻ. Ta có
(𝑛! − 1)
𝑛
+ 1 ≡ (−1)
𝑛
+ 1 ≡ 0 (mod 𝑛!) . (*)
Suy ra nếu 𝑛 là hợp số và 𝑑 là một ước nguyên tố của 𝑛 thì ta có

𝑛!
𝑑
− 1

𝑛
+ 1 =
𝑛

𝑘=1
𝐶
𝑘
𝑛
(𝑛!)
𝑘
𝑑
𝑘
≡ 0 (mod 𝑛!)
bởi vì rõ ràng 𝑑
2
| 𝑛! nên mỗi số hạng của tổng trên đều chia hết cho 𝑛!. Do vậy
trường hợp này cũng không thỏa mãn.
+, Xét 𝑛 là số nguyên tố lẻ và giả sử 𝑎 là số nguyên thỏa mãn 0 < 𝑎 ≤ 𝑛!, 𝑎

𝑛
+1
chia hết cho 𝑛!. Do 𝑛 lẻ nên ta có khai triển
𝑎
𝑛
+ 1 = (𝑎 + 1)

𝑎
𝑛−1
− 𝑎
𝑛−2
+ · · · + 𝑎
2
− 𝑎 + 1

.
Ta sẽ chứng minh 𝑎 = 𝑛! − 1 là giá trị duy nhất thỏa mãn.
Xét 𝑝 là số nguyên tố, 𝑝 ≤ 𝑛. Ta có 𝑝 | 𝑎
𝑛
+ 1 ⇒ 𝑝 | (−𝑎)
𝑛
− 1. Theo định lý
Fermat ta lại có 𝑝 | (−𝑎)
𝑝−1
− 1 nên 𝑝 | (−𝑎)
(𝑛,𝑝−1)
− 1 = −𝑎 − 1. Suy ra 𝑛 | 𝑎 + 1
(𝑝 = 𝑛) và nếu 𝑝 < 𝑛 thì 𝑎
𝑛−1
− 𝑎

𝑛−2
+ · · · + 𝑎
2
− 𝑎 + 1 ≡ 𝑛 (mod 𝑝) ̸≡ 0 (mod 𝑝)
do (𝑛, 𝑝) = 1. Từ đó ta thu được 𝑎
𝑛−1
− 𝑎
𝑛−2
+ · · · + 𝑎
2
− 𝑎+ 1 và (𝑛 − 1)! nguyên
tố cùng nhau, suy ra (𝑛 − 1)! | 𝑎 + 1. Mà 𝑛 | 𝑎 + 1 nên 𝑛! | 𝑎 + 1. Do điều kiện
0 < 𝑎 ≤ 𝑛! ⇒ 𝑛! = 𝑎 + 1 ⇒ 𝑎 = 𝑛! − 1. Cùng với (*) ta có 𝑎 = 𝑛! − 1 là giá trị
duy nhất thỏa mãn.
Vậy tất các số nguyên dương 𝑛 cần tìm là tất cả các số nguyên tố. 
Bài 6. (Dự tuyển IMO 2006) Chứng minh rằng với mọi số nguyên dương 𝑛 luôn
tồn tại số nguyên 𝑚 sao cho 2
𝑚
+ 𝑚 chia hết cho 𝑛.
Lời giải.
Ta sẽ chứng minh bằng quy nạp theo 𝑛 rằng luôn tồn tại số nguyên dương 𝑚
đủ lớn để 2
𝑚
≡ −𝑚 (mod 𝑛) .
+, Với 𝑛 = 1 hiển nhiên đúng.
+, Xét 𝑛 > 1. Theo tính chất của hàm Euler ta có dãy các số mũ của 2 theo
mô-đun 𝑛 là tuần hoàn với chu kỳ là bội của 𝜙 (𝑛). Do đó với 𝑥, 𝑦 đủ lớn và
𝑥 ≡ 𝑦 (mod 𝜙 (𝑛)) thì 2
𝑥
≡ 2

𝑦
(mod 𝑛).
Chọn số 𝑚 có dạng 𝑚 ≡ −2
𝑘
(mod 𝑛𝜙 (𝑛)). Khi đó 2
𝑚
≡ −𝑚 (mod 𝑛) ⇔
2
𝑚
≡ 2
𝑘
(mod 𝑛). Theo giả thiết quy nạp ta có thể chọn được số 𝑘 đủ lớn
để 2
𝑘
≡ −𝑘 (mod 𝜙 (𝑛)) ⇒ −2
𝑘
≡ 𝑚 ≡ 𝑘 (mod 𝜙 (𝑛)). Từ đó ta thu được
2
𝑚
≡ −𝑚 (mod 𝑛). Theo nguyên lý quy nạp ta có điều phải chứng minh. 
Bài 7. (CĐT Mỹ 2007) Hỏi có tồn tại hay không hai số nguyên dương 𝑎, 𝑏 sao
cho 𝑎 không là ước của 𝑏
𝑛
− 𝑛 với mọi số nguyên dương 𝑛.
11
www.VNMATH.com
Lời giải.
Trước hết ta chứng minh rằng với mọi số nguyên dương 𝑎, 𝑏 thì với 𝑛 đủ lớn
ta có
𝑏

𝑛+𝜙(𝑎)
≡ 𝑏
𝑛
(mod 𝑎) . (*)
Thật vậy, giả sử 𝑎 = 𝑝
𝛼
1
1
𝑝
𝛼
2
2
. . . 𝑝
𝛼
𝑘
𝑘
, trong đó 𝑝
1
, 𝑝
2
, , 𝑝
𝑘
là các số nguyên tố phân
biệt. Vì 𝜙 là hàm nhân tính nên ta có
𝜙 (𝑎) = 𝜙 (𝑝
𝛼
1
1
) 𝜙 (𝑝
𝛼

2
2
) . . . 𝜙 (𝑝
𝛼
𝑘
𝑘
) =

𝑝
𝛼
1
1
− 𝑝
𝛼
1
−1
1

𝑝
𝛼
2
2
− 𝑝
𝛼
2
−1
2

. . .


𝑝
𝛼
𝑘
𝑘
− 𝑝
𝛼
𝑘
−1
𝑘

= 𝑎

1 −
1
𝑝
1

1 −
1
𝑝
2

· · ·

1 −
1
𝑝
𝑘

.

Khi đó ta có 𝜙 (𝑝
𝛼
𝑖
𝑖
) | 𝜙 (𝑎) và 𝜙 (𝑎) < 𝑎. Với mỗi 𝑝
𝑖
, ta có hai trường hợp:
+, Nếu 𝑝
𝑖
là ước của 𝑏 ⇒ 𝑏
𝑛
≡ 0 (mod 𝑝
𝛼
𝑖
𝑖
) với 𝑛 ≥ 𝛼
𝑖
+ 1. Suy ra 𝑏
𝑛+𝜙(𝑎)

𝑏
𝑛
𝑏
𝜙(𝑎)
≡ 𝑏
𝑛
≡ 0 (mod 𝑝
𝛼
𝑖
𝑖

) với 𝑛 ≥ 𝛼
𝑖
+ 1.
+, Nếu 𝑝
𝑖
không là ước của 𝑏 ⇒ gcd (𝑝
𝛼
𝑖
𝑖
, 𝑏) = 1. Theo định lý Euler thì
𝑏
𝜙
(
𝑝
𝛼
𝑖
𝑖
)
≡ 1 (mod 𝑝
𝛼
𝑖
𝑖
), mà 𝜙 (𝑝
𝛼
𝑖
𝑖
) | 𝜙 (𝑎) ⇒ 𝑏
𝑛+𝜙(𝑎)
≡ 𝑏
𝑛

(mod 𝑝
𝛼
𝑖
𝑖
) .
Tức là với mỗi 𝑝
𝑖
ta luôn chọn được 𝑛
𝑖
sao cho với mọi 𝑛 > 𝑛
𝑖
thì 𝑏
𝑛+𝜙(𝑎)

𝑏
𝑛
(mod 𝑝
𝛼
𝑖
𝑖
). Do đó nếu ta chọn 𝑁 = max {𝑛
𝑖
} khi đó với mọi 𝑛 > 𝑁, ta có
𝑏
𝑛+𝜙(𝑎)
≡ 𝑏
𝑛
(mod 𝑝
𝛼
𝑖

𝑖
) với mọi 1 ≤ 𝑖 ≤ 𝑘. Bởi vì các 𝑝
𝑖
là các số nguyên tố phân
biệt nên 𝑏
𝑛+𝜙(𝑎)
≡ 𝑏
𝑛
(mod 𝑎) với mọi 𝑛 > 𝑁 (đpcm).
Trở lại bài toán, ta sẽ chỉ ra bằng quy nạp theo 𝑎 rằng với mọi số nguyên
dương 𝑎, 𝑏 thì luôn tồn tại vô hạn số nguyên dương 𝑛 sao cho 𝑎 là ước của 𝑏
𝑛
− 𝑛
(**).
+, Với 𝑎 = 1 rõ ràng khẳng định (**) đúng.
+, Giả sử (*) đúng với mọi số nguyên 1 ≤ 𝑎 < 𝑎
0
(𝑎
0
≥ 2). Do 𝜙 (𝑎) < 𝑎 nên
theo giả thiết quy nạp và theo (*) suy ra tồn tại vô số số nguyên dương 𝑛 sao cho
𝜙 (𝑎) | (𝑏
𝑛
− 𝑛) và 𝑏
𝑛+𝜙(𝑎)
≡ 𝑏
𝑛
(mod 𝑎) .
Với mỗi 𝑛 như vậy, đặt
𝑡 =

𝑏
𝑛
− 𝑛
𝜙 (𝑎)
và 𝑛
1
= 𝑏
𝑛
= 𝑛 + 𝑡𝜙 (𝑎) .
Thế thì theo định lý Euler
𝑏
𝑛
1
− 𝑛
1
≡ 𝑏
𝑛+𝑡𝜙(𝑎)
− 𝑛 − 𝑡𝜙 (𝑎) ≡ 𝑏
𝑛
− 𝑛 − 𝑡𝜙 (𝑎) ≡ 0 (mod 𝑎) ,
tức là 𝑛
1
= 𝑏
𝑛
thỏa mãn yêu cầu. Theo giả thiết quy nạp, có vô hạn số 𝑛
1
= 𝑏
𝑛
thỏa mãn điều kiện trong khẳng định (**), từ đó (**) đúng. Và do đó không có
các số nguyên dương 𝑎, 𝑏 nào thỏa mãn đầu bài. 

Bài 8. (CĐT Mỹ 2003) Tìm tất cả các bộ ba số nguyên tố (𝑝, 𝑞, 𝑟) thỏa mãn
𝑝 | 𝑞
𝑟
+ 1, 𝑞 | 𝑟
𝑝
+ 1, 𝑟 | 𝑝
𝑞
+ 1.
12
www.VNMATH.com
Lời giải.
Giả sử 𝑝, 𝑞, 𝑟 thỏa mãn đầu bài suy ra chúng là ba số nguyên tố phân biệt.
Trường hợp 1 : Cả ba số 𝑝, 𝑞, 𝑟 đều lẻ. Ta có 𝑝 | 𝑞
𝑟
+ 1 nên theo ví dụ 10 ta có
2𝑟 | 𝑝 − 1 hoặc 𝑝 | 𝑞
2
− 1.
+, Nếu 2𝑟 | 𝑝 − 1 ⇒ 𝑝 ≡ 1 (mod 𝑟) ⇒ 0 ≡ 𝑝
𝑞
+ 1 ≡ 2 (mod 𝑟) mà 𝑟 > 2 nên
mâu thuẫn với đầu bài.
+, Nếu 𝑝 | 𝑞
2
− 1 = (𝑞 − 1) (𝑞 + 1). Hơn nữa do 𝑝 lẻ và 𝑝 − 1, 𝑝 + 1 là các số
chẵn nên ta có
𝑝 |
𝑞 − 1
2
hoặc 𝑝 |

𝑞 + 1
2
.
Khi đó ta luôn có 𝑝 ≤
𝑞 + 1
2
< 𝑞. Bằng cách chứng minh tương tự ta cũng có
𝑞 < 𝑟, 𝑟 < 𝑝. Ta cũng có mâu thuẫn nên trường hợp này không xảy ra được.
Trường hợp 2 : Một trong ba số 𝑝, 𝑞, 𝑟 phải bằng 2, do tính xoay vòng nên ta
có thể giả sử 𝑝 = 2. Khi đó 𝑟 | 2
𝑞
+ 1, lại theo ví dụ 10 ta có 2𝑞 | 𝑟 − 1 hoặc
𝑟 | 2
2
− 1.
Rõ ràng nếu 2𝑞 | 𝑟 − 1 như trường hợp trên ta cũng có 𝑟 ≡ 1 (mod 𝑞) ⇒ 0 ≡
𝑟
𝑝
+ 1 ≡ 2 (mod 𝑞) ⇒ 𝑞 = 2 ta có mâu thuẫn vì 𝑝, 𝑞, 𝑟 phân biệt. Do đó 𝑟 | 2
2
− 1
suy ra 𝑟 = 3 và 𝑞 | 𝑟
2
+ 1 = 10 ⇒ 𝑞 = 5. (thỏa mãn).
Vậy các bộ cần tìm là các hoán vị xoay vòng của (2, 5, 3). 
Bài 9. (Iran 2007) Cho 𝑛 là số nguyên dương và 𝑛 = 2
2007
𝑘 +1 với 𝑘 là số nguyên
lẻ. Chứng minh rằng
𝑛  2

𝑛−1
+ 1.
Lời giải.
Giả sử 𝑛 | 2
𝑛−1
+ 1 =

2
𝑘

2
2007
+ 1. Gọi 𝑝 là một số ước nguyên tố tùy ý của 𝑛
và đặt 𝑡 = 2
𝑘
, 𝑑 = ord
𝑝
(𝑡). Ta có 𝑡
2
2007
≡ −1 (mod 𝑝) ⇒ 𝑡
2
2008
≡ 1 (mod 𝑝). Suy
ra 𝑑 | 2
2008
, 𝑑  2
2007
, tức là 𝑑 = 2
2008

mà 𝑑 | 𝑝 − 1 ⇒ 𝑝 ≡ 1 (mod 2
2008
). Do đó
𝑛 ≡ 1

mod 2
2008

⇒ 2
2007
𝑘 ≡ 0

mod 2
2008

⇒ 2 | 𝑘.
Điều này mâu thuẫn với giả thiết, ta có kết luận của bài toán. 
Bài 10. (Việt Nam 2001) Cho 𝑛 là số nguyên dương và 𝑎, 𝑏 là các số nguyên tố
cùng nhau lớn hơn 1. Giả sử 𝑝, 𝑞 là hai ước lẻ lớn hơn 1 của 𝑎
6
𝑛
+ 𝑏
6
𝑛
. Tìm số dư
trong phép chia của 𝑎
6
𝑛
+ 𝑏
6

𝑛
cho 6.12
𝑛
.
Lời giải.
Trước hết ta chứng minh 𝑝 ≡ 1 ≡ 𝑞 (mod 2
𝑛+1
). Vì (𝑎, 𝑏) = 1 suy ra (𝑎, 𝑝) =
(𝑏, 𝑝) = 1. Gọi 𝑏

là nghịch đảo của 𝑏 mô-đun 𝑝 và đặt 𝐴 = 𝑎𝑏

. Khi đó theo giả
thiết
𝑝 | 𝑎
6
𝑛
+ 𝑏
6
𝑛
| 𝐴
6
𝑛
+ 1 | 𝐴
2.6
𝑛
− 1.
13
www.VNMATH.com
Suy ra ord

𝑝
(𝐴) | 2.6
𝑛
, ord
𝑝
(𝐴) | 6
𝑛
. Do đó ta suy ra ord
𝑝
(𝐴) = 2
𝑛+1
3
𝑛

ord
𝑝
(𝐴) | 𝑝 − 1 ⇒ 𝑝 ≡ 1 (mod 2
𝑛+1
). Chứng minh tương tự cho 𝑞. Vậy 𝑝 ≡
𝑞 ≡ 1 (mod 2
𝑛+1
) ⇒ 𝑝
6
𝑛
≡ 𝑞
6
𝑛
≡ 6
𝑛
.2

𝑛+1
+ 1

mod (2
𝑛+1
)
2

(theo khai triển
Newton). Từ đó 𝑝
6
𝑛
≡ 𝑞
6
𝑛
≡ 1 (mod 2
2𝑛+1
). (*)
Hơn nữa ta có 𝜙 (6
𝑛+1
) = 2.6
𝑛
⇒ 𝑝
6
𝑛
≡ ±1 (mod 6
𝑛+1
). Giả sử 𝑝
6
𝑛


−1 (mod 6
𝑛+1
) suy ra 𝑝
6
𝑛
≡ −1 (mod 2
𝑛+1
) mà từ (*) ta có 𝑝
6
𝑛
≡ 1 (mod 2
𝑛+1
) ⇒
2𝑝
6
𝑛
≡ 0 (mo d 2
𝑛+1
) mâu thuẫn vì 𝑝 lẻ. Do đó ta thu được 𝑝
6
𝑛
≡ 1 (mo d 2
2𝑛+1
)
và 𝑝
6
𝑛
≡ 1 (mod 6
𝑛+1

) từ đó 𝑝
6
𝑛
≡ 1 (mod 6.12
𝑛
). Tương tự 𝑞
6
𝑛
≡ 1 (mod 6.12
𝑛
).
Suy ra số dư cần tìm là 2. 
Bài 11. (Trung Quốc 2009) Tìm tất cả các cặp số nguyên tố (𝑝, 𝑞) sao cho
𝑝𝑞 | 5
𝑝
+ 5
𝑞
.
Lời giải.
Rõ ràng cặp (5, 5) thỏa mãn. Xét trường hợp 𝑝 = 5, 𝑞 ̸= 5 ta có 5𝑞 | 5
5
+ 5
𝑞

𝑞 | 625 + 5
𝑞−1
. Theo định lý Fermat suy ra 𝑞 là ước của 626, trong trường hợp
này ta có nghiệm (5, 313) và (5, 2). Tương tự trường hợp 𝑞 = 5, 𝑝 ̸= 5 ta có hai
nghiệm (313, 5) và (2, 5).
Xét 𝑝 = 2, 𝑞 ̸= 5 ta có 2𝑞 | 25 + 5

𝑞
⇒ 25 + 5
𝑞
≡ 0 (mod 2𝑞) và theo định
lý Fermat ta thu được 30 ≡ 0 (mod 2𝑞) từ đó dễ thấy 𝑞 = 3 thỏa mãn. Do đó
trong trường hợp trong hai số 𝑝, 𝑞 có một số bằng 2 và số còn lại khác 5 ta có hai
nghiệm (2, 3) và (3, 2).
Xét 𝑝, 𝑞 là hai số khác 2 và 5. Ta có 5
𝑝
+ 5
𝑞
≡ 0 (mod 𝑝𝑞) ⇒ 5 + 5
𝑞

0 (mod 𝑝) ⇒ 5
𝑞−1
≡ −1 (mod 𝑝) ⇒ 5
2(𝑞−1)
≡ 1 (mod 𝑝). Tương tự ta có
5
𝑝−1
≡ −1 (mod 𝑞). Chọn 𝑘 sao cho 2
𝑘
‖ ord
𝑝
5 mà ord
𝑝
5 | 𝑝 − 1, và ord
𝑝
5 |

2 (𝑞 − 1) ; ord
𝑝
5  (𝑞 − 1). Suy ra 2
𝑘−1
‖ 𝑞 − 1, tức là số mũ lớn nhất của 2 mà
là ước của 𝑝 − 1 lớn hơn số mũ lớn nhất của 2 mà là ước của 𝑞 − 1. Do tính đối
xứng nên ta cũng có số mũ lớn nhất của 2 mà là ước của 𝑞 − 1 lớn hơn số mũ lớn
nhất của 2 mà là ước của 𝑝 − 1. Đây là điều mâu thuẫn, trong trường hợp này
bài toán vô nghiệm.
Vậy các nghiệm của bài toán là (2, 3) ; (2, 5) ; (5, 5) ; (5, 313) và các nghiệm đối
xứng của nó là (3, 2) ; (5, 2) ; (313, 5). 
Bài 12. (CĐT Romanian 2000) Cho 𝑎 > 1 là số nguyên dương. Tìm số nguyên
dương 𝑛 nhỏ nhất sao cho 2
2000
| 𝑎
𝑛
− 1.
Lời giải.
Ta có 𝑎
𝑛
≡ 1 (mod 2
2000
). Theo định lý Euler suy ra 𝑛 | 𝜙 (2
2000
) = 2
1999
⇒ 𝑛
là lũy thừa của 2. Đặt 𝑛 = 2
𝑘
với 𝑘 ≥ 0. Khi đó ta có

𝑎
𝑛
− 1 = (𝑎 − 1) (𝑎 + 1)

𝑎
2
+ 1

𝑎
4
+ 1

· · ·

𝑎
2
𝑘−1
+ 1

.
Mà 𝑎
2𝑚
+ 1 ≡ 2 (mod 4) nên 2
𝑘
− 1 ‖ (𝑎
2
+ 1) (𝑎
4
+ 1) · · ·


𝑎
2
𝑘−1
+ 1

. Do đó
ta cần tìm 𝑘 nhỏ nhất để 2
2001−𝑘
| (𝑎 − 1) (𝑎 + 1) = 𝑎
2
− 1. Vì thế nếu đặt
14
www.VNMATH.com
𝑚 = 𝑣
2
(𝑎
2
− 1) (tức là số mũ của 2 lớn nhất của 𝑎
2
− 1) ta có
𝑘 =

0 nếu 𝑚 ≥ 2001
2001 − 𝑚 nếu 𝑚 < 2001.
Vậy 𝑛 = 2
𝑘
với 𝑘 xác định theo công thức trên. 
Bài 13. (CĐT Romanian 2005) Giải phương trình trong tập số nguyên dương
3
𝑥

= 2
𝑥
𝑦 + 1.
Lời giải.
Bằng quy nạp ta dễ dàng chứng minh được hai nhận xét sau:
∘ Với mọi số nguyên dương 𝑛 ta có 2
𝑛+2
‖ 3
2
𝑛
− 1,
∘ Với mọi số nguyên dương 𝑛 ≥ 3 ta có 2
𝑛
> 𝑛 + 2.
Viết lại phương trình đã cho dưới dạng
2
𝑥
𝑦 = (3 − 1)

3
𝑥−1
+ 3
𝑥−2
+ · · · + 3 + 1

= 2

3
𝑥−1
+ 3

𝑥−2
+ · · · + 3 + 1

.
Đặt 𝑥 = 2
𝑛
𝛼 với 𝛼, 𝑛 ∈ N và 𝛼 lẻ.
+, Nếu 𝑛 = 0 tức là 𝑥 là số lẻ, mà 3
𝑥−1
+ 3
𝑥−2
+ + 3 + 1 là tổng của 𝑥 số
lẻ nên là số lẻ, suy ra 2 ‖ 2
𝑥
𝑦 ⇒ 𝑥 = 1 ⇒ 𝑦 = 1. Từ đó (𝑥, 𝑦) = (1, 1) là một
nghiệm của phương trình.
+, Nếu 𝑛 ≥ 1, ta có

3
2
𝑛

2
2
𝑛
𝛼
𝑦 =

3
2

𝑛

𝛼
− 1
𝛼
=

3
2
𝑛
− 1



3
2
𝑛

𝛼−1
+

3
2
𝑛

𝛼−2
+ · · · +

3
2

𝑛

+ 1

.


3
2
𝑛

𝛼−1
+

3
2
𝑛

𝛼−2
+ · · · +

3
2
𝑛

+ 1 là tổng của 𝑛 số lẻ nên là số lẻ. Do đó
theo nhận xét trên ta có 2
𝑛+2
‖ 3
𝑥

− 1 = 2
2
𝑛
𝛼
𝑦. Suy ra 𝑛 + 2 ≥ 2
𝑛
𝛼 ⇒ 𝑛 ≤ 2.
Kiểm tra trực tiếp dễ thấy với 𝑛 = 1 ⇒ 𝛼 = 1 ⇒ 𝑥 = 2, 𝑦 = 2; với 𝑛 = 2 tương
tự ta có 𝛼 = 1 và từ đó 𝑥 = 4, 𝑦 = 5.
Vậy phương trình có các nghiệm (1, 1) ; (2, 2) và (4, 5). 
Bài 14. (Dự tuyển IMO 1991) Tìm số nguyên dương 𝑘 lơn nhất sao cho
1991
𝑘
| 1990
1991
1992
+ 1992
1991
1990
.
Lời giải.
Trước hết ta chứng minh bằng quy nạp rằng với mọi số lẻ 𝑎 ≥ 3 và số nguyên
𝑛 ≥ 0 thì ta có
𝑎
𝑛+1
‖ (𝑎 + 1)
𝑎
𝑛
− 1 và 𝑎
𝑛+1

‖ (𝑎 − 1)
𝑎
𝑛
− 1.
Rõ ràng 𝑛 = 0 hiển nhiên đúng. Giả sử (𝑎 + 1)
𝑎
𝑛
= 1 + 𝑁𝑎
𝑛+1
, 𝑎  𝑁. Khi đó
(𝑎 + 1)
𝑎
𝑛+1
=

1 + 𝑁𝑎
𝑛+1

𝑎
= 1 + 𝑎.𝑁𝑎
𝑛+1
+ 𝐶
2
𝑎
𝑁
2
𝑎
2𝑛+2
+ 𝑀𝑎
3𝑛+3

.
15
www.VNMATH.com
Mà 𝑎 lẻ nên 𝐶
2
𝑎
chia hết cho 𝑎. Suy ra (𝑎 + 1)
𝑎
𝑛+1
= 1 + 𝑁

𝑎
𝑛+2
với 𝑎  𝑁

.
Do đó ta có ngay
1991
1993
‖ 1990
1991
1992
+ 1, 1991
1991
‖ 1992
1991
1990
− 1.
Vậy 𝑘 = 1991. 
Bài 15. (IMO 1990) Tìm tất cả các số nguyên dương 𝑛 sao cho

2
𝑛
+ 1
𝑛
2
là số
nguyên.
Lời giải
Với 𝑛 = 1 thỏa mãn. Xét 𝑛 > 1 ⇒ 𝑛 lẻ. Giả sử 𝑝 ≥ 3 là ước nguyên tố
nhỏ nhất của 𝑛. Ta có gcd (𝑝 − 1, 𝑛) = 1 và 𝑝 | 2
𝑛
+ 1 | 2
2𝑛
− 1. Theo định
lý Fermat 𝑝 | 2
𝑝−1
− 1 suy ra 𝑝 | gcd (2
𝑝−1
− 1, 2
2𝑛
− 1) = 2
gcd(2𝑛,𝑝−1)
− 1. Mà
gcd (2𝑛, 𝑝 − 1) ≤ 2 suy ra 𝑝 | 3 ⇒ 𝑝 = 3. Đặt 𝑛 = 3
3
𝑑 với 2, 3  𝑑.
Ta có nhận xét: Nếu 2
𝑚
− 1 chia hết cho 3
𝑟

thì 𝑚 chia hết cho 3
𝑟−1
(xem ví
dụ 3). Hơn nữa do 3
2𝑘
| 𝑛
2
| 2
2𝑛
− 1 ⇒ 3
2𝑘−1
| 𝑛 = 3
𝑘
𝑑 ⇒ 𝑘 = 1.
Giả sử 𝑑 > 1 và 𝑞 là ước nguyên tố nhỏ nhất của 𝑑. Rõ ràng 𝑞 lẻ và 𝑞 ≥
5, gcd (𝑛, 𝑞 − 1) | 3. Khi đó ta có 𝑞 | 2
2𝑛
− 1, 𝑞 | 2
𝑞−1
− 1 ⇒ 𝑞 | 2
gcd(2𝑛,𝑞−1)
− 1 |
2
6
− 1 = 3
2
.7. Do đó 𝑞 = 7 ⇒ 7 | 𝑛 | 2
𝑛
+ 1 mà 2
𝑛

+ 1 ≡ 2, 3, 5 (mod 7) mâu
thuẫn. Vậy 𝑑 = 1 ⇒ 𝑛 = 3 thỏa mãn. Đáp số 𝑛 = 1, 𝑛 = 3. 
Bài 16. (CĐT Trung Quốc 2005) Cho 𝑏, 𝑚, 𝑛 là các số nguyên dương 𝑏 > 1, 𝑚 ̸=
𝑛. Chứng minh rằng nếu 𝑏
𝑚
− 1 và 𝑏
𝑛
− 1 có cùng các ước nguyên tố thì 𝑏 + 1 là
lũy thừa của 2.
Lời giải.
Trước hết ta chứng minh bài toán cho trường hợp 𝑛 = 1, tức là nếu 𝑎 > 1, 𝑘 >
1 và 𝑎
𝑘
− 1 và 𝑎 − 1 có cùng các nhân tử nguyên tố thì 𝑘 và 𝑎 + 1 là lũy thừa
của 2. Phản chứng, giả sử 𝑝 là ước nguyên tố lẻ của 𝑘. Khi đó mọi ước nguyên
tố 𝑞 của 1 + 𝑎 + 𝑎
2
+ · · · + 𝑎
𝑝−1
là ước của 1 + 𝑎 + 𝑎
2
+ · · · + 𝑎
𝑘−1
| 𝑎
𝑘
− 1 | 𝑎 − 1.
Suy ra 1 + 𝑎 + 𝑎
2
+ · · · + 𝑎
𝑝−1

≡ 𝑝 (mod 𝑞) ≡ 0 (mod 𝑞), nên 𝑝 = 𝑞. Do đó
1 + 𝑎 + 𝑎
2
+ · · · + 𝑎
𝑝−1
= 𝑝
𝑡
, 𝑡 > 1. Đặt 𝑎 = 𝑢𝑝 + 1 ⇒ 1 + 𝑎 + 𝑎
2
+ · · · + 𝑎
𝑝−1

𝑝 (mod 𝑝
2
) mâu thuẫn. Vậy 𝑘 là lũy thừa của 2. Gọi 𝑟 là ước nguyên tố bất kỳ
của 𝑎 + 1 ⇒ 𝑟 | 𝑎
𝑘
− 1 ⇒ 𝑟 | 𝑎 − 1 ⇒ 𝑟 = 2.
Trong trường hợp 𝑛 bất kỳ, đặt 𝑑 = gcd (𝑚, 𝑛) , 𝑘 = 𝑚/𝑑. Ta thu được 𝑏
𝑘𝑏
− 1
và 𝑏
𝑑
− 1 có cùng các ước nguyên tố. Do đó theo chứng minh trên ta có 𝑘 là lũy
thừa của 2 và 𝑏
𝑑
+ 1 = 2
𝑡
, 𝑡 ≥ 2. Xét mô-đun 4 suy ra 𝑑 lẻ. Giả sử 𝑟 là ước nguyên
tố của 𝑏 + 1 suy ra 𝑟 | 𝑏 + 1 | 𝑏

2
− 1 | 𝑏
𝑘𝑑
− 1 ⇒ 𝑟 | 𝑏
𝑑
− 1 ≡ −2 (mod 𝑟). Từ đó
𝑟 = 2, tức là 𝑏 + 1 là lũy thừa của 2. 
III. Một số bài tập
Bài 1. (Dự tuyển IMO 2007) Cho 𝑏, 𝑛 là các số nguyên. Giả sử với mỗi 𝑘 > 1
luôn tồn tại số nguyên 𝑎
𝑘
sao cho 𝑏 − 𝑎
𝑛
𝑘
chia hết cho 𝑘. Chứng minh rằng 𝑏 = 𝐴
𝑛
với 𝐴 là số nguyên nào đó.
16
www.VNMATH.com
Hướng dẫn: Biểu diễn 𝑏 = 𝑝
𝛼
1
1
. . . 𝑝
𝛼
𝑙
𝑙
. Do 𝑏
2
| 𝑏 − 𝑎

𝑛
𝑏
2
⇒ 𝑝
𝛼
𝑖
𝑖
‖ 𝑎
𝑛
𝑏
2
∀𝑖. Suy ra
𝑛 | 𝛼
𝑖
∀𝑖 tức là 𝑏 là lũy thừa đúng bậc 𝑛.
Bài 2. (Dự tuyển IMO 2002) Cho 𝑝
1
, 𝑝
2
, , 𝑝
𝑛
là các số nguyên tố phân biệt lớn
hơn 3. Chứng minh rằng 2
𝑝
1
𝑝
2
𝑝
𝑛
+ 1 có ít nhất 4

𝑛
ước số.
Hướng dẫn: Chú ý nếu 𝑢, 𝑣 lẻ và (𝑢, 𝑣) = 1 thì ước chung lớn nhất của 2
𝑢
+ 1
và 2
𝑣
+ 1 là 3. Từ đó 2
𝑢𝑣
+ 1 chia hết cho (2
𝑢
+ 1) (2
𝑣
+ 1) /3. Sau đó chứng minh
bài toán bằng quy nạp theo 𝑛.
Bài 3. (IMO 2006) Xác định tất cả các cặp số nguyên (𝑥, 𝑦) thỏa mãn phương
trình
1 + 2
𝑥
+ 2
2𝑥+1
= 𝑦
2
.
Hướng dẫn: Xét 𝑥 > 0, 𝑦 > 0 và 𝑦 lẻ. Chú ý 2
𝑥
(1 + 2
𝑥+1
) = (𝑦 − 1) (𝑦 + 1) từ
đó ta có thể biểu diễn 𝑦 theo 𝑥. Suy ra 𝑥 = 4, 𝑦 = 23. Đáp số (0, ±2) ; (4, ±23) .

Bài 4. (Dự tuyển IMO 2001) Cho 𝑝 là số nguyên tố lớn hơn 3. Chứng minh rằng
tồn tại số nguyên 𝑎 với 1 ≤ 𝑎 ≤ 𝑝 − 2 sao cho hoặc 𝑎
𝑝−1
− 1 hoặc (𝑎 + 1)
𝑝−1
− 1
chia hết cho 𝑝
2
.
Hướng dẫn: Đặt 𝐴 = {1 ≤ 𝑎 ≤ 𝑝 − 1 : 𝑎
𝑝−1
̸≡ 1 (mod 𝑝
2
)}. Vì ít nhất một
trong hai số 𝑎, 𝑝 − 𝑎 thuộc 𝐴 và 1 /∈ 𝐴 và |𝐴| ≥ (𝑝 − 1) /2 = 𝑘 và giả sử có
đúng một trong hai số 2𝑖, 2𝑖 + 1 (1 ≤ 𝑖 ≤ 𝑘 − 1) thuộc 𝐴. Xét 𝑝 ≥ 7 ⇒ 2𝑘 − 1 =
𝑝 − 2 /∈ 𝐴 ⇒ 𝑝 − 3 = 2𝑘 − 2 ∈ 𝐴. Bằng cách xét mô-đun 𝑝
2
⇒ 2𝑘 − 3 ∈ 𝐴, tức là
𝑝 − 3, 𝑝 − 4 thỏa mãn.
Bài 5. (CĐT Việt Nam 2008) Cho 𝑚, 𝑛 là các số nguyên dương. Chứng minh
rằng (2𝑚 + 3)
𝑛
+ 1 chia hết cho 6𝑚 khi và chỉ khi 3
𝑛
+ 1 chia hết cho 4𝑚.
Hướng dẫn: Khai triển Newton, bài toán trở thành chứng minh 2𝑚 | 3
𝑛
+ 1
và 3 | (2𝑚)

𝑛
+ 1 khi và chỉ khi 4𝑚 | 3
𝑛
+ 1.
Bài 6. (Dự tuyển IMO 2000) Xác định tất cả các bộ ba số nguyên dương (𝑎, 𝑚, 𝑛)
sao cho 𝑎
𝑚
+ 1 là ước của (𝑎 + 1)
𝑛
.
Hướng dẫn: Chú ý nếu 𝑢 | 𝑣
𝑙
⇒ 𝑢 | (gcd (𝑢, 𝑣))
𝑙
. Xét 𝑎 > 1 và 𝑚 > 1,
khi đó 𝑚 lẻ. Giả sử 𝑝 là ước nguyên tố của 𝑚. Bằng cách xét mô-đun, suy ra
(𝑏
𝑝
+ 1) / (𝑏 + 1) = 𝑝 ⇒ 𝑝 = 3. Đáp số: {(1, 𝑚, 𝑛) ; (𝑎, 1, 𝑛) ; (2, 3, 𝑛 ≥ 2)} .
Bài 7. (Nga 2000) Hỏi có tồn tại ba số 𝑎, 𝑏, 𝑐 > 1 đôi một nguyên tố cùng nhau
và thỏa mãn
𝑏 | 2
𝑎
+ 1, 𝑐 | 2
𝑏
+ 1, 𝑎 | 2
𝑐
+ 1.
Hướng dẫn: Ta có 𝑎, 𝑏, 𝑐 là lẻ. Giả sử 𝑎 = min {𝑎, 𝑏, 𝑐}. Nếu 𝑎, 𝑏, 𝑐 nguyên tố
suy ra 𝑎 = 3, không tồn tại 𝑏, 𝑐. Trường hợp bất kỳ ta ký hiệu 𝜋 (𝑛) là ước nguyên

tố nhỏ nhất của 𝑛. Khi đó nếu 𝑝 là số nguyên tố sao cho 𝑝 | 2
𝑦
+ 1 và 𝑝 < 𝜋 (3)
thì 𝑝 = 3. Sử dụng tính chất này suy ra mâu thuẫn.
Bài 8. (IMO 2000) Hỏi có tồn tại hay không số nguyên dương 𝑛 có đúng 2000
ước nguyên tố và 2
𝑛
+ 1 chia hết cho 𝑛.
Hướng dẫn: Sử dụng quy nạp theo số các ước số, 𝑘. Chỉ ra rằng ta có thể tìm
𝑛 với đúng 𝑘 ước nguyên tố sao cho 𝑛 | (2
𝑛
+ 1) và có một ước nguyên tố của
2
𝑛
+ 1 nhưng không là ước của 𝑛.
17
www.VNMATH.com
Bài 9. (CĐT Iran 2006) Cho 𝑛 là số nguyên dương. Tìm tất cả các bộ 𝑛 số
𝑎
1
, 𝑎
2
, , 𝑎
𝑛
đôi một phân biệt và đôi một nguyên tố cùng nhau sao cho với mọi
1 ≤ 𝑖 ≤ 𝑛 ta có
𝑎
1
+ 𝑎
2

+ · · · + 𝑎
𝑛
| 𝑎
𝑖
1
+ 𝑎
𝑖
2
+ · · · + 𝑎
𝑖
𝑛
.
Hướng dẫn: Chứng minh 𝑎
1
+𝑎
2
+· · ·+𝑎
𝑛
| 𝑛 (𝑛 − 1) và hai số này bằng nhau.
Nếu 𝑛 ≥ 8 thì 𝑎
1
+ 𝑎
2
+ · · · + 𝑎
𝑛
≥ 1 + 2 + 3 + 5 + · · · + 2𝑛 + 1 = 𝑛
2
− 7 > 𝑛
2
− 𝑛.

Đáp số 𝑛 = 1, 𝑎
1
tùy ý.
Bài 10. (Iran 2004) Cho 𝑎
1
, 𝑎
2
, , 𝑎
𝑛
là các số nguyên không đồng thời bằng
nhau. Chứng minh rằng tồn tại vô số số nguyên tố 𝑝 sao cho có tồn tại số nguyên
dương 𝑘 sao cho
𝑝 | 𝑎
𝑘
1
+ 𝑎
𝑘
2
+ · · · + 𝑎
𝑘
𝑛
.
Hướng dẫn: Ta có thể giả sử gcd (𝑎
1
, 𝑎
2
, , 𝑎
𝑛
) = 1 và 𝑎
𝑛

≥ 2. Giả sử chỉ có
hữu hạn các số nguyên tố 𝑝
1
, , 𝑝
𝑚
. Chọn 𝑘 thích hợp để suy ra 𝑎
𝑘
1
+ 𝑎
𝑘
2
+ · · · +𝑎
𝑘
𝑛
có ước nguyên tố lớn hơn 𝑝
1
𝑝
2
𝑝
𝑚
.
Bài 11. (Dự tuyển IMO 2002) Tìm tất cả các cặp số nguyên 𝑚, 𝑛 ≥ 3 sao cho
tồn tại vô hạn các số nguyên 𝑎 thỏa mãn
𝑎
𝑚
+ 𝑎 − 1
𝑎
𝑛
+ 𝑎
2

− 1
là một số nguyên.
Hướng dẫn: Rõ ràng 𝑛 < 𝑚. Sử dụng tính chất đa thức suy ra biểu thức 𝐴 đã
cho nhận giá trị nguyên với mọi 𝑎. Đa thức ở tử số chia hết đa thức ở mẫu số nên
chúng có nghiệm chung 𝛼 ∈ (0, 1) từ đó đánh giá theo 𝛼 suy ra 𝑚 < 2𝑛. Cuối
cùng chọn 𝑎 = 2 và xét mô-đun 𝑑 = 2
𝑛
+ 3 (mẫu số) ta được (𝑚, 𝑛) = (5, 3) .
Bài 12. (Trung Quốc 2008) Tìm tất cả các bộ ba số (𝑝, 𝑞, 𝑛) sao cho 𝑞
𝑛+2

3
𝑛+2
(mod 𝑝
𝑛
) , 𝑝
𝑛+2
≡ 3
𝑛+2
(mod 𝑞
𝑛
) trong đó 𝑝, 𝑞 là các số nguyên tố lẻ và 𝑛 là
số nguyên dương.
Hướng dẫn: Nếu hai trong ba số 𝑝, 𝑞, 3 bằng nhau ta có nghiệm (3, 3, 𝑛). Xét
trường hợp chúng phân biệt và 𝑝 > 𝑞. Ta có 𝑝
𝑛
𝑞
𝑛
| 𝑝
𝑛+2

+ 𝑞
𝑛+2
− 3
𝑛+2
< 2𝑝
𝑛+2

𝑝
2
> 𝑞
𝑛−1
. Mà 𝑞
𝑛+2
− 3
𝑛+2
≥ 𝑝
𝑛
. Suy ra 𝑛 ≤ 3.
18
www.VNMATH.com

×